Jump to content
Urch Forums

shilpishrestha

1st Level
  • Posts

    466
  • Joined

Everything posted by shilpishrestha

  1. Natib: for 2nd statement take an example: 8 -- 1,2,4,8 6 -- 1,2,3,6 4 -- 1.2,4 3 -- 1,3 i was also thinking the same way....but if 2 is already a factor of the number than scenario changes.
  2. Answer given by Kaplan: 2. A. (1) Sufficient. Since the quantity 2R is divisible by 3, one of those two factors must be divisible by 3. Since 1 isn't; R must be. (2) Insufficient. We know that quantity 3R is evenly divisible by 3, which means that at least one of the factors must be divisible by 3. The problem, though, is that 3 is evenly divisible by 3, making it impossible for us to determine if R is. But i am not convinced with the answer. 1. when 2R is evenly dividible by 3, means 2R can be: 6, 12, 18, 24, 30 So R can be: 3, 6, 12, 15 How com R is evenly divisible by 3. Please explain..... 2. 3R can be: 6, 12, 18, 24, 30 R can be: 2, 4, 6, 8, 10
  3. We have to support the claim that company should announce the new device after the sales of old device have begun to decline. * New technology often becomes less expensive after an initial surge in sales. --> If a technology becomes less expensive after certain point of time, then buyers would buy the same old technology. Instead of buying the new technology, which may be expensive. So, this choice doesn't support the claim. * Media outlets, such as television programs and magazines, often report on the planned introduction of new devices while the sales of old devices are still strong. --> this choice is strongly supporting the claim that media, TV all play a major role in making people aware of the introduction of new technology in the market that may stop people from buying the old product. * Many consumers are unable to determine whether new technology is superior to current technology. --> does not support the conclusion. * Surveys have shown that some consumers make only one or two technology purchases per year, whereas others make more frequent purchases. --> Out of scope also too weak to support anything. * Consumers tend to be loyal to technology companies whose products they enjoy using. --> this choice does not do anyhting, out of scope.
  4. 808 Bob Explanation: D is awkward and confusing in that it has two modifying phrases ("carrying as many as seven passengers" and "compared with most sport utility vehicles") between the subject "minivans" and the first verb "cost". Reading the sentence as choice D phrases it, when I get to "compared" I first think it's the verb. Only when I get to "cost" do I realize that the phrase beginning with "compared" is not the predicate but a second modifier. D is confusing also in that it is not clear in what way minivans are being compared with most SUVs. Is the fact that minivans carry as many as seven passengers one of the differences? There is absolutely nothing wrong with choice A. It tells us two things about minivans: Minivans carry as many as seven passengers. Minivans (compared with most SUVs) cost less, etc. Since these two facts have the same subject, we can put them together, without repeating the subject, and inserting "and" between the verbs: Minivans carry as many as seven passengers and (compared with most SUVs) cost less, etc. It would be wrong to put a comma in front of "and". On the other hand, the comma after "and" makes perfect sense: it's one of a pair of commas setting off the modifying phrase "compared with most SUVs". Beware those OAs! They are not always correct!
  5. I also went for B. Please help us understand how D is better than B?
  6. confused between B and D. but yes "their" has no clear referent in choice D. So will go for B.
  7. Agree with amitabh. should be C. ksgill, if we combine and solve the above two statements, we wil get that y has to be greater than 0. so x > 0. as mentioned by amitabh above.
  8. yes should be B. same explanation as given by flex_many
  9. Agree with OA. Is SQRT [(x - 3) ^2] = 3-x ? (1) x does not equal 3 (2) - x |x| > 0 |x-3| = 3-x if (x-3) x 1st, Insufficient 2nd, sufficient
  10. gud explanation by ankitarish. Agree with E.
  11. Really a very good, thought provoking CR. Although i chose A, now i agree with C. Good explanation by cubicle.
  12. Yes it should be C. It is a very strong sentence, the idea which is not talked in the sentence.
  13. Here conclusion is that to increase the profit margins the company's head must find new source of the raw material. we have weaken this conslusion. i.e. it is not profitable to find new raw material source. A. New competitors have entered the market every six months for the past two years, resulting in price wars that have progressively driven down revenues across the market. --> does not weaken the conclusion. B. Although the earthquake occurred two years ago, the region's mines have still not recovered to pre-earthquake production capacity. --> it does not strongly support that finding the new source is the not gud idea. C. There are several other regions in the world where the raw material is mined, but those regions do not produce as much of the raw material as the current source region. --> hmmm looks gud, says this region is better than the others. D. The company could use a completely different raw material to make its product. --> Out of scope E. Recent advances in mining technology will make mining the raw material much more efficient and cost-effective in the future --> this choice does not weaken the concusion that finding the new region is a gud idea. So, IMO C What is the OA?
  14. Hi David, My profile is: 1st GMAT score: 530 2nd GMAT score: 590 ( M 48, V 24, AWA 4.5 ) Xth: 82.2 (2nd in the class) XII: 85.6 (1st in the school) Btech from UPTech CS: 77.2 (3rd in the class) One paper accepted in a big US conference. Two years of work Ex in Amdocs India Pvt Ltd. I am looking to apply to MIS programs in US. Please suggest, What should I do?
  15. 1. the number has to be a multiple of 24, because it is a divisor of both 8 and 3. 2. take an example as 12, both 4 and 6 are divisors of 12. But still it is not a multiple of 24. So can't determine. Answer is A.
  16. If it is x, then answer is C, And if it is n, then answer should be E. right?
  17. Conclusion: Investors looking for profit prospects in oil industry should invest in paper industry, from which they can use by product for oil manufacturing. (A) A small quantity of lignins are currently sold by paper manufacturers to chemical companies, but most of the lignins produced are burnt as waste. out of scope, we have to prove that the deal is not gud for investors. Investors doesn't come into picture in this choice. (B) The 20-dollar-a-barrel oil price as a threshold of profitability for using lignins allows for the increased cost of refining crude oil that has been extracted using lignins. looks gud © Only one-half to two-thirds of the total oil in a well can be extracted using conventional techniques of pumping and flooding with water. out of scope. (D) Petroleum-based substances that can be used as a substitute for lignins in extracting oil are costly and are made from oil, and these substances therefore increase in price as oil increases in price. i think its strenthening the argument. (E) The quantity of lignins produced annually in the manufacture of paper is several times larger than the amount that is likely to be useful in the oil industry. may be But i think B is stronger than E. Will go with B.
  18. Gud question. I chose C, but that answer should be A. Because C changes the meaning.
  19. IMO should be A. A is parallel and moreover, what does itself signify in B?
×
×
  • Create New...